An outbreak of MRSA infections is occurring in a hospital ward. The best mechanism for reducing further transmission is encouraging regular hand washing among ward staff, as hand washing is the most effective way to control the spread of MRSA in a hospital setting. Screening staff and treating patients are not as effective as hand washing for reducing transmission.
We take content rights seriously. If you suspect this is your content, claim it here.
Available Formats
Download as PDF, TXT or read online on Scribd
0 ratings0% found this document useful (0 votes)
83 views215 pages
1173
An outbreak of MRSA infections is occurring in a hospital ward. The best mechanism for reducing further transmission is encouraging regular hand washing among ward staff, as hand washing is the most effective way to control the spread of MRSA in a hospital setting. Screening staff and treating patients are not as effective as hand washing for reducing transmission.
We take content rights seriously. If you suspect this is your content, claim it here.
Available Formats
Download as PDF, TXT or read online on Scribd
You are on page 1/ 215
INTERESTING
MCQs IN PEDIATRICS
Consultant of Pediatric Emergency Medicine
One of the surgical wards notes an outbreak of Methicillin-resistant Staph. aureus (MRSA) infections. What is the best mechanism for reducing further transmission of this infection? A) Cleaning the floors and walls of the ward with chlorhexidine B) Close the ward for one month C) Encourage regular hand washing by ward staff D) Screen ward staff using nasal swabs and exclude those with positive cultures for MRSA E) Treatment of culture-positive patients with vancomycin Answer: C Cross-infection via hands of medical and nursing staff is a very important vehicle of transmission of MRSA. Hand washing before and after contact with patients is the single most effective measure to control hospital spread of this organism. Screening of ward staff is appropriate only in certain situations and should not be carried out unless recommended by the hospital infection control team. Vancomycin should never be used for MRSA decolonization. The hospital infection control policy should outline which patients should be screened and when decolonization should be attempted. A full-term newborn infant is having episodes of cyanosis and apnea, which are worse when he is attempting to feed, but he seems better when he is crying. The most important next step to quickly establish the diagnosis is a. Echocardiogram b. Ventilation perfusion scan c. Passage of catheter into nose d. Hemoglobin electrophoresis e. Bronchoscopic evaluation of palate and larynx'' Answer: C It is important to make the diagnosis of “Choanal Atresia” quickly because it responds to treatment but can be lethal if unrecognized and untreated. Most neonates are obligate nose breathers because they cannot breathe adequately through their mouths. Infants with choanal atresia have increased breathing difficulty during feeding and sleeping and improved respirations when crying. A variety of temporizing measures to maintain an open airway have been used, including oropharyngeal airways, positioning, tongue fixation, and endotracheal intubation, but surgical correction with placement of nasal tubes is most effective. The diagnosis can be made by failure to pass a catheter through the nose to the pharynx. A newborn is noted to be quite jaundiced at 3 days of age. Which of the following factors is associated with an increased risk of neurologic damage in a jaundiced newborn? a) Metabolic alkalosis b) Increased attachment of bilirubin to binding sites caused by drugs such as sulfisoxazole c) Hyperalbuminemia d) Neonatal sepsis e) Maternal ingestion of phenobarbital during pregnancy. Answer: d Neonatal sepsis Significant unconjugated serum bilirubin levels in full-term newborn infants can lead to diffusion of bilirubin into brain tissue and to neurologic damage. Sulfisoxazole and other drugs compete with bilirubin for binding sites on albumin; therefore, the presence of these drugs can cause dislocation, not increased affinity, of bilirubin to tissues. Metabolic acidosis also reduces binding of bilirubin, and neonatal sepsis interrupts the blood-brain barrier, thus allowing diffusion of bilirubin into the brain. Administration of phenobarbital has been used to induce glucuronyl transferase in newborn infants and can reduce, rather than exacerbate, neonatal jaundice. Other factors that reduce the amount of unconjugated bilirubin bound to albumin (and therefore cause an increase in free unconjugated bilirubin) include hypoalbuminemia and certain compounds (e.g., non-esterified fatty acids, which are elevated during cold stress) that compete with bilirubin for albumin binding sites. A 17 year old male from India presents with fever of 4 months duration and splenomegaly. What is the most likely diagnosis? A. Coccidiomycosis B. Giardiasis C. Tropical sprue D. Typhoid E. Visceral leishmaniasis Answer: E Visceral leishmaniasis (Kala-azar): is an endemic disease in several regions of India and sub-Saharan Africa. It is caused by the parasite Leishmania donovani and spread by Phlebotomus sand-flies. Leishmaniasis is common in immune- suppressed patients, particularly those infected with HIV. There has recently been a substantial increase of cases in the Mediterranean region. It has been estimated that 15% of HIV positive drug users in Spain are infected with Leishmania donovani infantum. Giardiasis and tropical sprue present with gastrointestinal symptoms and malabsorption. Typhoid is an acute illness. Coccidiomycosis is largely confined to the Americas. Most patients present with pulmonary symptoms although disseminated disease can occur particularly in the immune-suppressed. Manifestations of the first stage of severe acute iron poisoning include (A) lethargy and gastrointestinal irritation (B) metabolic alkalosis and hypertension (C) hemolysis and neutropenia (D) renal, hepatic, and cardiac failure (E) hypoglycemia and hepatic injury. Answer: (A) The clinical manifestations of iron poisoning have been organized into four stages. Stage I: is characterized by GI (vomiting, diarrhea, abdominal pain, and GI bleeding) and neurologic (lethargy or coma) signs. Stage II: of deceptive quiescence, of up to 48 hours, Stage III: characterized by shock and metabolic acidosis, with or without evidence of hepatic injury. Leukocytosis is common. Stage IV: (Late sequelae) include pyloric or antral stenosis and hepatic cirrhosis. A 4-m-old infant was taken to the pediatric casualty due to persistent vomiting. On examination the child was noted to be irritable. In the history it was noted that the preparation of infant formula had been in a lead-soldered bowl bought by the father on a recent travel to the Middle East. The lead levels in the venous blood were of 76 mug/dl (toxic level is above 10 mug/dl) Treatment should include a) Diphenhydramine b) Calcium disodium ethylene diamine tetraacetic acid c) Deferoxamine mesylate d) Flumazenil e) Folic acid Answer: B Childhood lead poisoning is characteristically a disease that occurs between the second and third years of life, generally resulting from the child's ingestion of lead- based paint or dust. However, lead poisoning may also appear in the first year of life. Intense acute exposure to lead also results in symptomatic poisoning. The characteristic symptoms and signs of acute lead toxicity include: abdominal colic, malaise, nausea, vomiting, constipation, fatigue, anemia, peripheral neuropathy, renal impairment, hepatic dysfunction, and alterations of the CNS. Lead poisoning in infancy may have unusual etiologies such as in utero transmission of lead by lead-poisoned women. Because sources of lead poisoning in infancy may be unusual, a detailed environmental investigation may be necessary to identify the exact source. Children exposed to lead in the first 2 years of life have a special vulnerability to the neurotoxicity of lead & developmental handicaps. The use of chelation in acute lead poisoning is guided by the patient's clinical status and blood lead levels. Any patient who is symptomatic with protracted GI problems or CNS toxicity from an acute lead exposure will require parenteral chelation therapy as an inpatient. Hospitalization and chelation is also indicated for any child with a blood lead level of greater than 69 mug/dl, regardless of whether the child is symptomatic. Patients admitted to the hospital should receive IV calcium disodium ethylene diamine tetra acetic acid (CaNa2 EDTA), the most efficient parenteral chelator of lead A pediatric patient treated for grand mal seizures develops abnormal values on liver function tests. Which of the following antiepileptic agents would cause this to occur? A. Carbamezine B. Valproic acid C. Phenytoin D. Phenobarbital E. Gabapentin. Answer: B Severe hepatotoxicity of an idiosyncratic nature is associated with valproic acid. The risk is very high in the pediatric population, particularly below the age of two. Fatalities generally occur within four months of treatment. Hepatotoxicity may be reversed in some individuals. A 5-month-old girl presented with history of constipation and delayed developmental milestones. She had prolonged physiological jaundice. On exam, she is hypoactive, has an open mouth with large tongue. Other systemic examinations are within normal. What is the next step in management? A) Checking T4 and TSH B) Checking serum bilirubin C) Doing CT scan of head D) Follow up after 4 weeks Answer: A Key points: Constipation, delayed developmental milestones, being hypoactive open mouth with large tongue, prolonged physiological jaundice = Hypothyroidism
(A): the right choice
Points with: manifestations of hypothyroidism. In any case, u've to investigate for treatable causes first. (B): there's no use of checking bilirubin + it is physiological. (C): there's no use of CT scan, even if there was brain damage, it's not treatable (D): the condition can’t be delayed A 12-year-old Chinese-American boy complains of gas, bloating, and diarrhea for the past 3 months. He has no nocturnal symptoms and denies weight loss. His recent history is significant only for a family trip to mainland China to visit his grandparents 2 months ago. Physical exam demonstrates a well-developed, well- nourished adolescent. The remainder of the examination is unremarkable, except for a non tender, tympanitic, slightly distended abdomen with active bowel sounds. His stool is watery, however occult blood test results are negative and the pH: 5.0 Of the following, the test that you are MOST likely to recommend is a A. 72-hour fecal fat B. Lactose breath test C. Lactulose breath test D. Small intestinal biopsy for disaccharidase levels E. Stool for ova and parasites Answer: B The boy described has a 3-month history of watery diarrhea in the absence of weight loss or nocturnal or other constitutional symptoms. Although a recent trip abroad suggests the possibility of an acquired enteric infection, his symptoms began before he left the United States. Examination of a stool specimen demonstrates an acidic pH of 5.0 and no occult blood loss. In this scenario, a diarrheal state occurring as a consequence of dietary causes, especially carbohydrate malabsorption. A previously healthy 6-m-old child who just completed antibiotic treatment for acute otitis media and now is healthy and asymptomatic undergoes a stool culture to check for antibiotic-associated diarrhea. The stool culture grows C.difficile. The preferred antibiotic treatment is: A-Oral clindamycin B-Oral vancomycin C-Intravenous vancomycin D-Oral metronidazole E-None of the above. Answer: E None of the above: antibiotic treatment is not indicated C.difficile is frequently isolated from the stool of healthy infants. The interpretation of a positive stool culture or toxin requires clinical correlation. Treatment would not be indicated for an asymptomatic infant. A 2 year old girl presented with a 12-hour history of fever and poor feeding. The family owned two cats. Her temperature was 39.5°C, and she was irritable, with no localizing signs or skin lesions. A full septic screen was performed. Cerebrospinal fluid (CSF) showed a neutrophilic pleocytosis and gram-negative coccobacilli. She was treated with IV cefotaxime and gentamicin. Within 24 hours both CSF and blood cultures showed growth of gram-negative bacilli. What is the likely infective organism? A. Legionella pneumophilia B. Haemophilus influenza C. Pasteurella multicoda D. E coli E. Pseudomonas aeruginosa. Answer: C Pasteurella multicoda. Pasteurella multicoda is an oral commensal of domestic pets known to be an opportunistic human pathogen after traumatic animal contact. The most common infections in humans are skin and pulmonary infections. Pasteurella meningitis occurs at extremes of age (infants), in the immunocompromised (associated with liver cirrhosis, renal disease and haematological malignancies) and after traumatic head injury. A 6-month-old child was noted to be normal at birth, but over the ensuing months you have been somewhat concerned about his slow weight gain and his mild delay in achieving developmental milestones. The family calls you urgently noting that their child seems unable to move the right side of his body. Which of the following conditions might explain this child’s condition? a. Phenylketonuria b. Homocystinuria c. Cystathioninuria d. Maple syrup urine disease e. Histidinemia. Answer: b Homocystinuria It can cause thromboembolic phenomena in the pulmonary and systemic arteries and particularly in the cerebral vasculature; vascular occlusive disease is in turn, one of the many causes of acute infantile hemiplegia. None of the other disorders listed is associated with acute hemiplegia. Phenylketonuria causes retardation and, on occasion, seizures; Maple syrup urine disease, an abnormality of the metabolism of leucine, leads to seizures and rapid deterioration of the central nervous system in newborn infants; Histidinemia is associated with speech impairments and other minor neurologic difficulties; Cystathioninuria is most likely a benign amino-aciduria with no effect on the central nervous system. The newborn pictured was born at home and has puffy, tense eyelids; red conjunctivae; a copious amount of purulent ocular discharge; and ecchymosis 2 days after birth. The most likely diagnosis is a. Dacryocystitis b. Chemical conjunctivitis c. Pneumococcal ophthalmia d. Gonococcal ophthalmia e. Chlamydial conjunctivitis. Answer: d Gonococcal Ophthalmia The time of onset of symptoms is somewhat helpful in the diagnosis of Ophthalmia Neonatorum. Gonococcal conjunctivitis has its onset within 2-5 days after birth and is the most serious of the bacterial infections. Prompt and aggressive topical treatment and systemic antibiotics are indicated to prevent serious complications such as corneal ulceration, perforation, and resulting blindness. Parents should be treated to avoid the risk to the child of reinfection. Chemical conjunctivitis is a self-limited condition that presents within 6-12 h of birth as a consequence of silver nitrate or erythromycin prophylaxis. Silver nitrate is believed by some to be ineffective prophylaxis against chlamydial conjunctivitis, which occurs 5-14 days after birth, but is no longer made for ocular prophylaxis. To avoid the risk of chlamydial pneumonia, treatment with systemic antibiotics is indicated for the infant as well as both parents in cases of chlamydial conjunctivitis. At 43 weeks’ gestation, a long, thin infant is delivered. The infant is apneic, limp, pale, and covered with “pea soup” amniotic fluid. The first step in the resuscitation of this infant at delivery should be a. Suction of the trachea under direct vision b. Artificial ventilation with bag and mask c. Artificial ventilation with endotracheal tube d. Administration of 100% oxygen by mask e. Catheterization of the umbilical vein. Answer: a Infants who are postmature (more than 42 weeks’ gestation) and show evidence of chronic placental insufficiency (low birth weight for GA and wasted appearance) have a higher-than-average chance of being asphyxiated, and passage of meconium into the amniotic fluid thus places these infants at risk for meconium aspiration. To prevent or minimize this risk, these infants should have immediate nasopharyngeal suction as their heads are delivered. Immediately after delivery and before initiation of respiration, their tracheas should be carefully and thoroughly suctioned through an endotracheal tube under direct vision with a laryngoscope. Afterward, appropriate resuscitative measures should be undertaken to establish adequate ventilation and circulation. Artificial ventilation performed before tracheal suction could force meconium into smaller airways. A 2-year-old boy develops bloody diarrhea shortly after eating in a fast food restaurant. A few days later, he develops pallor and lethargy; his face looks swollen and his mother reports that he has been urinating very little. Laboratory evaluation reveals low hematocrit and platelet count and positive blood and protein in the urine. Which of the following diagnoses is likely to explain these symptoms? a. Henoch-Schönlein purpura b. IgA nephropathy c. Intussusception d. Meckel diverticulum e. Hemolytic-uremic syndrome. Answer: e Hemolytic-uremic syndrome: is Triad of: Acute micro-angiopathic hemolytic anemia, Thrombocytopenia from increased platelet utilization, Renal insufficiency from vascular endothelial injury and local fibrin deposition. Ischemic changes result in renal cortical necrosis and damage to other organs such as colon, liver, heart, brain, and adrenal. Laboratory findings associated with HUS include Low hemoglobin level, decreased platelet count, hypoalbuminemia, and evidence of hemolysis on peripheral smear (burr cells, helmet cells, schistocytes). Urinalysis reveals hematuria and proteinuria. A marked reduction of renal function leads to oliguria and rising levels of blood urea nitrogen (BUN) and creatinine. Gastrointestinal bleeding and obstruction, ascites, Central nervous system findings such as somnolence, convulsions, and coma can occur. In the past decade, infection by the verotoxin-producing E.coli 0157:H7 has been implicated as a cause of hemolytic-uremic syndrome. This organism is epizootic in cattle. Outbreaks associated with undercooked contaminated hamburgers have been reported in several states. Roast beef, cow’s milk, and fresh apple cider have been implicated as well. The Coombs test is not positive in this type of hemolytic anemia. A 6-year-old boy has brown urine and healing impetigo lesions. He presents with hypertension, dyspnea, periorbital edema, and hepatomegaly. The most likely cause of his problem is a. IgA nephropathy b. Poststreptococcal glomerulonephritis c. Idiopathic hypercalciuria d. Pyelonephritis e. Sexually transmitted disease. Answer: b Acute post-streptococcal glomerulonephritis follows a skin or throat infection with certain nephritogenic strains of group A, β-hemolytic streptococci. Hematuria often colors the urine dark, and decreased urinary output can result in circulatory congestion from volume overload with pulmonary edema, periorbital edema, tachycardia, and hepatomegaly. This can be avoided by fluid restriction. Acute hypertension is common and can be associated with headache, vomiting, and encephalopathy with seizures. Pyelonephritis and sexually transmitted diseases can cause bloody urine, but rarely the other symptoms of impetigo as described in the question. IgA nephropathy is rare in black children and rarely presents with hypertension. Idiopathic hypercalciuria can present with blood and renal stones, but the other findings are distinctly unusual. A 2-year-old boy with undescended testes is referred to pediatric surgeon. Surgical correction will probably eliminate the risk of a. Testicular malignancy b. Decreased sperm count c. Torsion of testes d. Urinary tract infection e. Epididymitis. Answer: C At 1 year of age, 0.7% of boys born at term still have cryptorchidism. In adults with cryptorchidism, the risk of testicular malignancy is much higher than in unaffected men. Orchiopexy does not eliminate this risk, but repositioning the testes makes them accessible for periodic examinations. Whether the testes are brought into the scrotum or not, the sperm count can be reduced. The failure of the testes to develop, and their subsequent atrophy, can be detected by 6 m of age. Torsion of the testis is a potential risk because of the excessive mobility of the undescended testis, this is prevented by Orchiopexy. A newborn infant has mild cyanosis, diaphoresis, poor peripheral pulses, hepatomegaly, and cardiomegaly. Respiratory rate is 60 breaths/min, and Ht.R is 250 beats/min. The child most likely has congestive heart failure caused by A large atrial septal defect and valvular pulmonic stenosis b. A ventricular septal defect and transposition of the great vessels c. Total anomalous pulmonary venous return d. Hypoplastic left heart syndrome e. Paroxysmal atrial tachycardia. Answer: e Congestive heart failure from any cause can result in mild cyanosis, even in the absence of a right to left shunt, and in poor peripheral pulses when cardiac output is low. Congestive heart failure from many causes can be associated with a rapid pulse rate (up to 200 beats per min). A pulse rate greater than 250 beats per min, however, should suggest the presence of a tachyarrhythmia. Common causes for supraventricular tachycardia includes Wolff-Parkinson-White (WPW) syndrome, congenital heart disease, and sympathomimetic drugs. In this patient, evaluation for WPW and cardiac abnormalities must be accomplished after the congestive heart failure from the increased heart rate is under control. An ill-appearing 2-week-old girl is brought to the emergency room. She is pale and dyspneic with a respiratory rate of 80 breaths/min. Ht. rate is 195 beats/min, heart sounds are distant, and a gallop is heard. There is cardiomegaly by x-ray. An echocardiogram demonstrates poor ventricular function, dilated ventricles, and dilation of the left atrium. An electrocardiogram shows ventricular depolarization complexes that have low voltage. The diagnosis suggested by this clinical picture is a. Myocarditis b. Endocardial fibroelastosis c. Pericarditis d. Aberrant left coronary artery arising from pulmonary artery e. Glycogen storage disease of the heart. Answer: a The findings of pallor, dyspnea, tachypnea, tachycardia & cardiomegaly are common in congestive heart failure regardless of the cause. The most common causes of myocarditis include adenovirus and coxsackievirus B, although many other viruses can cause this condition. The constellation of findings in the question point to myocarditis as the etiology of this patient’s condition. The lack of echocardiographic findings other than ventricular and left atrial dilatation and poor ventricular function is inconsistent with both glycogen storage disease of the heart, in which there is muscle thickening, and pericarditis, since there is no pericardial effusion. It is also not consistent with an aberrant origin of the left coronary artery, although the origin of the coronary arteries can be more easily missed. On electrocardiogram, the voltages of the ventricular complexes seen with aberrant origin of the left coronary artery are not diminished, and a pattern of myocardial infarction can be seen. Voltages from the left ventricle are usually high in endocardial fibroelastosis, and both right and left ventricular forces are high in glycogen storage disease of the heart. A 2-year-old boy is brought into the emergency room with a C/O fever for 6 days and development of a limp. On examination, he is found to have an erythematous macular exanthem over his body, ocular conjunctivitis, dry and cracked lips, a red throat, and cervical LN. There is a grade II/VI vibratory systolic ejection murmur at the lower left sternal border. A white blood cell count and differential show predominant neutrophils with increased platelets on smear. The most likely diagnosis is a. Scarlet fever b. Rheumatic fever c. Kawasaki disease d. Juvenile rheumatoid arthritis e. Infectious mononucleosis. Answer: c Kawasaki disease (KD) Many conditions can be associated with prolonged fever, a limp caused by arthralgia, exanthem, adenopathy, and pharyngitis. Conjunctivitis, however, is suggestive of Kawasaki disease. The fissured lips, although common in KD, could occur after a long period of fever from any cause if the child became dehydrated. The predominance of neutrophils and high ESR are common to all. An increase in platelets within this constellation of symptoms, however, is found only in KD. KD presents as prolonged fever, rash, epidermal peeling on the hands and feet (especially around the fingertips), conjunctivitis, lymphadenopathy, fissured lips, oropharyngeal mucosal erythema, and arthralgia or arthritis. The diagnosis is still possible in the absence of one or two of these physical findings. Coronary artery aneurysms can develop, as can aneurysms in other areas. Initial treatment is typically IVIG and high-dose aspirin. The child will usually defervesce shortly after the infusion. Aspirin is typically kept at a higher dose until the platelet count begins to decrease, and then is continued at a lower dose for several weeks. While bacterial infection is in the differential diagnosis for this patient’s presentation and blood cultures are usually part of the evaluation, IV vancomycin should be reserved. A 9-month-old infant accidentally ingests an unknown quantity of digitalis. The most important non cardiac manifestation of toxicity in this infant is a. Fever b. Dizziness c. Vomiting d. Visual disturbances e. Urticaria. Answer: c An important non cardiac manifestation of digitalis toxicity in infants is vomiting. Affected infants also exhibit ECG changes, including sinus arrhythmia (bradycardia) and a wandering pacemaker, paroxysmal tachycardia, and a heart rate of less than 100 beats per min. The commonly used digitalis preparation in infants is digoxin. Digoxin blood levels of 2 ng/dL or less are usually therapeutic in adults; in contrast, therapeutic digoxin blood levels in infants range from 1-5 ng/dL, but the benefit of the higher levels in infants is doubtful. An infant weighing 1400 g (3 lb) is born at 32 weeks’ gestation in a delivery room that has an ambient temperature of 24°C (75°F). If left in an open crib for a few minutes, this child is likely to demonstrate a. Ruddy complexion b. Shivering c. Hypertension d. Increased respiratory rate e. Metabolic alkalosis. Answer: d A room temperature of 24°C (75°F) provides a cold environment for newborn infants. Aside from the fact that these infants emerge from a warm, 37.6°C (99.5°F) intrauterine environment, at birth, infants (and especially preterm infants) are wet, have a relatively large surface area for their weight, and have little subcutaneous fat. Within minutes of delivery, the infants are likely to become pale or blue and their body temperatures will drop. In order to bring body temperature back to normal, they must increase their metabolic rate; ventilation, in turn, must increase proportionally to ensure an adequate oxygen supply. Because a preterm infant is likely to have respiratory problems and be unable to oxygenate adequately, lactate can accumulate and lead to a metabolic acidosis. Infants rarely shiver in response to a need to increase heat production. A full-term infant is born after a normal pregnancy; delivery, however, is complicated by marginal placental separation. At 12 h of age, the child, although appearing to be in good health, passes a bloody meconium stool. For determining the cause of the bleeding, which of the following diagnostic procedures should be performed first? a. A barium enema b. An Apt test c. Gastric lavage with normal saline d. An upper gastrointestinal series e. A platelet count, prothrombin time, and partial thromboplastin time. Answer: b Apt test Hematemesis and melena are not uncommon in the neonatal period, especially if gross placental bleeding has occurred at the time of delivery. The diagnostic procedure that should be done first is the Apt test, which differentiates fetal from adult hemoglobin in a bloody specimen. If the blood in an affected infant’s gastric contents or stool is maternal in origin, further workup of the infant is obviated. Which of the following is true of Koplik's spots? 1) Are diagnostic of Measles 2) Located opposite the incisor teeth. 3) Only appear when fever is over 39°C 4) They appear as red papules on the plamar surface of the hands 5) Typically appear two days after the rash. Answer: 1 Are diagnostic of Measles. Koplik spots are a prodromic viral enanthem of measles manifesting two days before the measles rash itself. They are characterized as clustered, white lesions on the buccal mucosa near each Stenson's duct (opposite the premolars) and are pathognomonic for measles. The textbook description of Koplik spots is ulcerated mucosal lesions marked by necrosis, neutrophilic exudate, and neovascularization. They are described as "grains of salt on a wet background" and often fade as the macular rash develops. A 9 month old boy presents with a 24 hr. history of fever and poor feeding. He has become less responsive and has developed a purple rash over his arms and trunk. No neonatal problems, No drugs, fully immunized, No F/H or social history of note. On examination temperature is 38.2°C, RR 50/min (mild recession, HR 150/min. Cool peripheries, with capillary refill time of 4 seconds. Non-blanching purple spots 3-27mm over the arms and trunk. What is the most likely diagnosis? 1-meningococcal infection 2-Thrombocytopaenia 3-Trauma/ child abuse 4-Vasculitis (HSP) 5-Viral infection Answer: 1 Meningococcal infection. The history is brief, with a sick, shocked child with purpura. The diagnosis is purpura fulminans, probably due to septicaemia. The 3 commonest organisms which cause this are meningococcus type B,A&C, pneumococcus and Haemophilus influenza type B. The incidence of Meningitis C and Hib have declined dramatically since the introduction of routine immunisations. A mother delivers a neonate with meconium staining and Apgar scores of 3 at 1 & 5 min of life. She had no prenatal care and the delivery was by emergency cesarean section for severe fetal bradycardia. Which of the following sequelae could be expected to develop in this intubated neonate with respiratory distress? a. Sustained rise in pulmonary arterial pressure b. Hyperactive bowel sounds c. Microcephaly with micrognathia d. Cataracts e. Thrombocytosis. Answer: a Sustained rise in pulmonary arterial pressure. The low Apgar scores, meconium staining, and ensuing RD suggest that asphyxia has occurred. During a period of asphyxia, the resulting hypoxemia, acidosis, and poor perfusion can damage a neonate’s brain, heart, kidney, liver, and lungs. The resulting clinical abnormalities include cerebral edema, irritability, seizures, cardiomegaly, heart failure, renal failure, poor liver function, DIC, and RD syndrome. There can be excessively high pulmonary arterial pressure at the same time systemic blood pressure begins to fall, resulting in a persistent right-to-left shunt across a patent ductus arteriosus or foramen ovale. Which of the following is associated with a poorer prognosis for persons presenting with meningococcal disease? A-Presence of petechiae for <12 hr B-Meningitis C-Thrombocytosis D-Leukocytosis E-Low circulating levels of tumor necrosis factor Answer: A Poor prognostic signs for meningococcal disease include: Hypotension and shock, Purpura fulminans, Seizures, Leukopenia, Thrombocytopenia, High circulating levels of endotoxin and tumor necrosis factor. The presence of petechiae for less than 12 hr before admission, hyperpyrexia, and the absence of meningitis reflect rapid clinical progression and poorer prognosis. Potential cardiac problems in patients with MPS include all of the following except: A-Coronary artery disease B-Mitral regurgitation C-Pericarditis D-Aortic valve disease E-Cardiomyopathy F-Endocarditis Answer: C Pericarditis Storage in endothelial cells can cause coronary artery disease and even myocardial infarction. Cardiomyopathy may be present in infancy and is associated with endocardial fibroelastosis. Valve disease necessitating valve replacement usually occurs in older children. Hurler disease is characterized by all of the following except: A- Hepatosplenomegaly B- Coarse facies C- Dysostosis multiplex D- X-linked inheritance E- Corneal clouding Answer: D X-linked inheritance All the mucopolysaccharidoses are inherited as autosomal recessive disorders except Hunter syndrome (MPS II), which is inherited as an X-linked trait. A 5-mo-old infant with gross motor delay, hypotonia, and cardiomegaly has a CK level of 860 IU/L. Of the following, which is the most likely diagnosis? A-Spinal muscular atrophy B-Hypothyroidism C-Prader-Willi syndrome D-Pompe disease E-Down syndrome Answer: D Pompe disease is a glycogen storage disease that affects the heart and skeletal muscle. Cardiac involvement causes heart failure and a shortened P-R interval on the ECG. Treatment with recombinant enzyme replacement has shown promise in the management of Pompe disease. A 3-mo-old previously healthy and well infant presents to an emergency room with a generalized seizure. The physical examination shows a temperature of 36.5°C, respiration rate of 50/min, and marked hepatomegaly, with the liver edge palpable down to the umbilicus. Laboratory findings show Na, 141 mEq/L; Cl, 95 mEq/L; K, 4.5 mEq/L; BUN, 12 mg/dL; Cr, 0.2 mg/dL; glucose, 25 mg/dL; CO2, 12 mEq/L; uric acid, 7.1 mg/dL; cholesterol, 270 mg/dL; and triglyceride, 550 mg/dL. The most likely diagnosis is: A-Hyperinsulinemia B-Adrenal insufficiency C-Growth hormone deficiency D-Glycogen storage disease. Answer: D The child most likely has Glycogen Storage Disease, type I, (glucose-6-phosphatase deficiency). During fasting from a viral illness or by sleeping longer between feeds, the metabolic defect becomes manifest as an inability to release glucose from glycogen. Hypoglycemia, lactic acidosis, ketosis, and hyperlipidemia are noted, in addition to high uric acid levels. A 17-yr-old boy has a history of recurrent episodes of exercise intolerance with muscle cramps. He is otherwise healthy and well. Which of the following is most likely to develop during these episodes in this patient? A-Myoglobinuria B-Abnormal liver function C-Aminoaciduria D-Cardiac arrhythmia E-Metabolic acidosis. Answer: A Myoglobinuria The history is compatible with muscle glycogenosis. With decreased ability to release glucose from muscle glycogen, myocyte injury occurs and places the patient at risk for myoglobinuria and renal failure. A 5-yr-old boy who lives in an urban area is hospitalized because of low grade fever, flaccid paralysis of both legs, sensory changes, and absent ankle deep tendon reflexes. The child received only two immunizations with OPV, at the ages of 2 and 6 mo. The most probable diagnosis is: A- Acute paralytic poliomyelitis due to wild poliovirus B- Paralysis due to nonpolio enteroviruses C- Vaccine-associated poliomyelitis D- Guillain-Barré syndrome E- Tick-bite paralysis Answer: D Guillain-Barré syndrome, or autoimmune peripheral neuropathy, is symmetric and involves motor nerves but to a less extent can also affect sensory nerves. Polio is an anterior horn cell disease and is purely motor. Which of the following features distinguishes paralytic polio from Guillain- Barré syndrome? A-Pleocytosis is uncommon in paralytic polio B-Paralysis is usually asymmetric in paralytic polio C-Sensory changes are common in paralytic polio D-Paralytic polio only occurs in unimmunized persons Answer: B Paralytic polio is characterized by aseptic meningitis accompanied by asymmetric flaccid paralysis without sensory loss. In Guillain-Barré syndrome, the paralysis is characteristically symmetric, and sensory changes (paresthesias) are common. Pleocytosis is common in polio, whereas the cerebrospinal fluid in Guillain-Barré syndrome usually shows only elevated protein and occasionally a few cells. Which of the following diseases may be associated with a severe acute reaction to radiotherapy? A. Chédiak-Higashi syndrome B. Neurofibromatosis C. Chronic mucocutaneous candidiasis D. Ataxia-telangiectasia E. Wiskott-Aldrich syndrome Answer: D A child with ataxia-telangiectasia (AT), because of the associated hereditary DNA repair defect, is unable to repair acute radiation damage. Conventional doses of radiotherapy to treat lymphomas (to which AT predisposes) have caused acute radiation sickness, and children have died. It is important to diagnose ataxia-telangiectasia in young patients with lymphoma to avoid radiotherapy. The other choices listed in the question are not associated with severe acute radiation reaction. A 2-yr-old boy presents with a peeling, erythematous rash on the hands and feet. The mother reports that he has become ill-tempered and refuses to walk about, preferring to lie in bed. Physical examination reveals an irritable, pale child with photophobia. Temperature is 98.5°F, heart rate is 80/min, and respiratory rate is 23/min. Tremor of the tongue is evident. Further history and follow-up evaluation reveal that the child's elder brothers have been playing with liquid mercury. Which of the following is the most likely diagnosis? A. Measles B. Fifth disease C. Kawasaki disease D. Acrodynia E. Photosensitivity Answer: D Acrodynia, characterized by erythemia, peeling, and neurologic signs, is characteristic of chronic mercury intoxication. Examination of the cerebrospinal fluid of an 8-year-old, mildly febrile child with nuchal rigidity and intermittent stupor shows the following: WBCs 100/μL (all lymphocytes), negative Gram stain, protein 150 mg/dL, and glucose 15 mg/dL. The most likely diagnosis is A. Tuberous sclerosis B. Tuberculous meningitis C. Stroke D. Acute bacterial meningitis E. Pseudotumor cerebri. Answer: B Included among those things that can cause the clinical picture are viral meningitis, TB meningitis, meningeal leukemia, and medulloblastoma, all of which can cause pleocytosis as well as elevated protein and lowered glucose concentrations in (CSF). Of the four diseases, tuberculous meningitis is associated with the lowest glucose levels in CSF. The cellular response to viral meningitis eventually will be predominantly lymphocytic. Cells found in the CSF of a child who has meningeal leukemia most commonly are lymphocytes or lymphoblasts. Children who have a medulloblastoma generally present with the signs and symptoms caused by a mass in the posterior cranial fossa; their pleocytotic CSF contains unusual-appearing cells of the monocytic variety. The decrease in the glucose concentration of CSF associated with these disorders has been attributed to a disturbance of glucose transport as a result of meningeal irritation. In pseudo tumor cerebri, the constituents of CSF are generally normal except for a low protein content in some instances. Acute bacterial disease typically causes PMN cells and positive Gram stains. Neither tuberous sclerosis nor stroke typically causes these findings on CFS examination. Acute bacterial disease typically causes PMN cells and positive Gram stains A 9-year-old girl is brought to her pediatrician with the complaint of severe, intermittent headaches for the last several months. The physical examination, including a careful neurologic examination, is normal. Which of the following characteristics support the diagnosis of childhood migraine? a. Strong family history of migraine b. Frequently isolated to the occipital region c. Frequently associated with attention deficit hyperactive disorder d. Duration of headache more than 24 h e. Persistence of headache after sleep. Answer: a In contrast to adults, children with migraine most often have “common” migraine: bifrontal headache without an aura or diffuse throbbing headache of only a few hours’ duration. As with adults, the headaches can be terminated with vomiting or sleep. Family history is frequently positive. Association with attention deficit hyperactive disorder is not common, but a relationship with seizure disorder can be seen. A 3-year-old child is unconscious, with symmetrically small pupils. These findings occur in which of the following? a. Pontine hemorrhage b. Phenylephrine overdose c. Trauma with resultant third nerve palsy d. Atropine-induced coma e. Cyclopentolate exposure. Answer: a Pinpoint pupils are found in coma, narcotics use, pilocarpine treatment, Horner syndrome, pontine hemorrhage, posterior synechia formation, pesticide or nerve gas exposure, and tertiary syphilis. In addition, trauma, third nerve paralysis, or closed-angle glaucoma can also cause pinpoint pupils. Dilated pupils are seen with drugs such as epinephrine, phenylephrine, topical antihistamines/vasoconstrictor combinations and atropine-like agents. A 4-year-old boy presents with a history of constipation since the age of 6 months. His stools, produced every 3 to 4 days, are described as large and hard. Physical examination is normal; rectal examination reveals a large ampulla, poor sphincter tone, and stool in the rectal vault. The plain film of his abdomen is shown. The next step in the management of this child would be a. Lower GI barium study b. Parental reassurance and dietary counseling c. Serum electrolyte measurement d. Upper GI barium study e. Initiation of synthroid Answer: b The radiograph demonstrates a stool-filled megacolon. Hirschsprung disease is usually suspected in the chronically constipated child despite the fact that 98% of such children have functional constipation. Finding a dilated, stool-filled anal canal with poor tone on the physical examination of a well grown child supports the diagnosis of functional constipation. The difficulty in treating functional constipation once it has been established emphasizes the need for prompt identification and treatment of problems with defecation and for counseling of parents regarding proper toileting behavior. The extensive workup of this patient would likely be negative and expensive, and is not indicated. A newborn infant is born with petechiae scattered across his body. His platelet count is noted to be 22,000/μL with a hemoglobin of 12 mg/dL. Which of the following is most likely to explain this infant’s condition? a. Congenital cytomegalovirus infection b. Uncomplicated prematurity c. Chlamydial conjunctivitis d. Maternal ingestion of aspirin e. Nasolacrimal duct stenosis. Answer: a Thrombocytopenia and hemolytic anemia, as seen in this patient, are common manifestations of the TORCH (toxoplasmosis, other, rubella, cytomegalovirus, and herpes simplex) infections as well as congenital syphilis. Both increased platelet destruction and impaired production of platelets may be the mechanisms involved. Some mothers who have had ITP and who have high levels of antiplatelet antibody in the maternal plasma can give birth to thrombocytopenic infants because of transplacental crossing of antiplatelet IgG antibody. The syndrome of congenital amegakaryocytic thrombocytopenia and bilateral absence of the radii (thrombocytopenia-absent radius, or TAR, syndrome) is a well-known entity. Maternal ingestion of aspirin can lead to bleeding in the newborn, not as a result of thrombocytopenia but as a consequence of transient impairment of the infant’s platelet aggregation. Neither chlamydial conjunctivitis nor nasolacrimal duct stenosis is usually associated with thrombocytopenia. A 15 year old boy presents with tremor of both hands. Over the previous months he has developed a mild dysarthria. He has a history of behavioral problems, of a depressive/psychotic nature. The most likely diagnosis is: 1 ) Alzheimer's disease 2 ) Huntington's disease 3 ) Neuroacanthocytosis 4 ) variant Creutzfeldt-Jakob disease 5 ) Wilson's disease. Answer: 5 Wilson's disease: is a rare disorder of copper metabolism which is inherited as an autosomal recessive disease. It is associated with extrapyramidal features (tremor, parkinsonism, dystonia), dysarthria, psychiatric features, cirrhosis and a deposit of brownish-green pigment around the margin of the cornea (Kayser-Fleischer ring). Variant Creutzfeldt-Jakob disease is characterized by myooclonus and rapid onset dementia. A mother brought in her previously health 5-month-old infant for evaluation of a rash on his head, face, neck, and trunk, including the genital area. She reported that the boy’s skin initially had developed some redness on and off for a few weeks. Over the last 3 to 4 days, however, the affected skin had begun to peel. She reported no fever or irritability, and that the boy had been eating fine, with normal wet diapers and normal stools. The infant had no symptoms of upper respiratory infection, and he had no sick contact or history of travel.On physical examination, the infant was vigorous, was in no acute distress, and was well hydrated. He had patches of yellow, greasy scales over his scalp and umbilical area, in addition to erythema and peeling of skin in the creases behind his ears and on his neck, upper chest, back, and genital area. There was no tenderness, increased warmth, or swelling, and no drainage from the lesions. What explains this infant’s dermatologic symptoms? A. Tinea corporis B. Seborrheic dermatitis C. Psoriasis D. Eczema Answer: B Seborrheic dermatitis: is a chronic, relapsing, inflammatory condition that can affect infants and adults alike. Like many other skin conditions, seborrheic dermatitis tends to worsen in the cold and dry winter months and improve during the summer months. Although its cause is not completely understood, it has a predilection for areas that are abundant in sebaceous glands, including the face, scalp, chest, and anogenital region. In susceptible persons, the inflammatory process may be mediated by fungal metabolites released from sebaceous triglycerides. Seborrheic dermatitis also tends to be more severe in immune-compromised individuals. Poorly defined, erythematous, scaling plaques characterize seborrheic dermatitis. The rash can involve one area or it can spread diffusely. It is commonly known as cradle cap, because scaling usually is present on the scalp. Other affected areas can include the face, ears, and neck. Erythema often is seen in the flexural folds and inter-triginous areas. The diagnosis is based on a detailed history and thorough physical examination. Biopsy usually is not indicated unless the diagnosis is uncertain. Differential diagnosis of seborrheic dermatitis includes psoriasis, rosacea, atopic dermatitis, tinea corporis, and rarer conditions such as secondary syphilis, systemic lupus erythematosus, candidiasis, and dermatomyositis. Treatment: Seborrheic dermatitis during infancy usually is self-limited, resolving within a few weeks to several months. The acute phase may be treated effectively with topical antifungals, corticosteroids, and/or other agents with nonspecific antimicrobial or anti-inflammatory properties. The intermittent use of topical antifungals helps prevent recurrence. Care should be taken to avoid using high- potency corticosteroids on the face and inter-triginous areas due to the possible adverse effect of skin atrophy. A three year old boy with known renal failure presents is brought to Casualty after a seizure. He is noted to have marked bowing of the long bones of his lower limbs. An x ray of the lower limbs shows cupping, fraying and irregularity of the metaphysis. These manifestations are most likely secondary to deficiency of a) Vitamin A b) Vitamin D2 c) alpha-Tocopherol d) Vitamin K e) Vitamin C Answer: B Rickets: is associated with abnormalities of growth including skeletal deformity, bone fractures and slipped epiphyses. Hypocalcemia may be severe enough to produce tetany, seizures, and laryngeal spasm though the commoner manifestations are listlessness and irritability. Children often are unable to walk without support as the disease progresses and muscle weakness is present. The classical skeletal deformities start by 6 months of age with frontal bossing. Later chest deformities include a lateral collapse of both chest walls (Harrison's sulcus) and rachitic rosary. If it goes untreated, progressive bony deformities result in bowing particularly in the tibia, femur, radius, and ulna. Fractures may also result. A 20-m-old child develops hemolytic anemia, anuria, azotemia, and thrombo- cytopenia after a bout of febrile bloody diarrhea. The most likely etiologic agent of this illness is: A- Campylobacter jejuni B- Salmonella typhi C- Entero-hemorrhagic Escherichia coli D- Aeromonas E- Non-typhi Salmonella Answer: C Entero-hemorrhagic Escherichia coli Shiga toxin-producing E. coli O157:H7 is an entero-hemorrhagic pathogen and is responsible for most episodes of hemolytic-uremic syndrome. Shigella dysenteriae is occasionally responsible for the hemolytic-uremic syndrome (HUS) A 10-mo-old child presents to the emergency department with a 4-day history of fever, with temperatures to 40.3°, and watery diarrhea, and has just experienced a generalized seizure. The most likely cause of this syndrome is: A- Salmonella gastroenteritis B- Aeromonas gastroenteritis C- Shigella gastroenteritis D- Rotavirus gastroenteritis E- Drug ingestion Answer: C Shigella gastroenteritis Shigella usually causes diarrhea and fever and sometimes, particularly in young infants, seizures can occur. The diarrhea may be watery and of large volume initially, evolving into frequent small-volume, bloody mucoid stools; most children (>50%) never progress to the stage of bloody diarrhea, whereas in others the 1st stools are bloody. Neurologic findings are among the most common extraintestinal manifestations of bacillary dysentery, occurring in as many as 40% of hospitalized infected children. Enteroinvasive E. coli can cause similar neurologic toxicity. Convulsions, headache, lethargy, confusion, nuchal rigidity, or hallucinations may be present before or after the onset of diarrhea. The cause of these neurologic findings is not understood. In the past, these symptoms were attributed to the neurotoxicity of Shiga toxin, but it is clear that this explanation is wrong since the organisms isolated from children with Shigella-related seizures are usually not Shiga toxin producers. Seizures sometimes occur when little fever is present, suggesting that simple febrile convulsions do not explain their appearance. Hypocalcemia or hyponatremia may be associated with seizures in a small number of patients. Although symptoms often suggest central nervous system infection, and cerebrospinal fluid pleocytosis with minimally elevated protein levels can occur, Shigella meningitis is rare. Polycythemia is associated with all of the following except: A-Trisomy 21 B-Donor twin in a twin-twin transfusion C-Neonatal Graves disease D-Adrenogenital syndrome E-Infant of diabetic mother Answer: B Donor twin in a twin-twin transfusion The recipient twin develops polycythemia. Additional risks for polycythemia include intrauterine growth retardation, postmature infants, Beckwith-Wiedemann syndrome, and being born at high altitude. All of the following tumors are associated with Epstein-Barr virus except: A-Burkitt lymphoma B-Kaposi sarcoma C-Leiomyosarcoma D-Lymphoproliferative disease in immunocompromised persons E-Nasopharyngeal carcinoma Answer: B Kaposi sarcoma EBV is associated with several malignancies, including nasopharyngeal carcinoma, Burkitt lymphoma, Hodgkin disease, and lymphoproliferative diseases and leiomyosarcomas in immune-compromised persons. Kaposi sarcoma is associated with HHV-8. A 9-day-old neonate, born by vaginal delivery to a 21-yr-old healthy woman, is presented with fever, lethargy, and poor feeding. There are no diagnostic findings on physical examination, and results of sepsis evaluation, including a CBC and cerebrospinal fluid studies, are unremarkable. Ampicillin and cefotaxime are begun. Two days later all bacterial cultures are negative but the child's clinical condition worsens, with falling blood pressure, decreased level of consciousness, thrombocytopenia, and elevated liver enzymes. The indicated change in treatment is: A-Addition of amphotericin B-Addition of vancomycin C-Addition of ribavirin D-Addition of acyclovir E-Substitution with meropenem and amikacin Answer: D Addition of acyclovir The history, inability to confirm bacterial infection, and worsening clinical condition are suggestive of neonatal herpes simplex virus infection. Acyclovir therapy should be initiated. Repeat cerebrospinal fluid analysis or MRI might also be recommended A 3-yr-old boy presents with a 7-day history of fever, cervical lymphadenopathy, foul breath, and painful oral lesions on his tongue, gums, and lips. For the past 3 days he has had a red, painful swollen area about the nail of his right thumb with an area of fluid by the nail bed, unresponsive to warm soaks and a first-generation cephalosporin. The most likely etiologic agent is: A-Staphylococcus aureus B-Mucocutaneous candidiasis C-Coxsackievirus D-Adenovirus E-Herpes simplex virus Answer: E Herpes simplex virus The child has herpetic gingiva-stomatitis. He has auto-inoculated his thumb by sucking, and herpetic whitlow has developed Recommended management for a mother with active genital HSV infection during labor is: A) Culture of blood from the newborn, with treatment based on culture results B) Culture of blood from the newborn, with empirical acyclovir therapy C) Intravenous acyclovir treatment for the mother D) Cesarean section within 4 hr of rupture of membranes E) Intravenous acyclovir treatment for the mother and cesarean section within 4 hr of rupture of membranes Answer: D Cesarean section within 4 hr of rupture of membranes Both the American Academy of Pediatrics and the American College of Obstetrics and Gynecology recommend cesarean section if primary, first-episode, or recurrent HSV lesions are present on the mother at the onset of labor. Only 15-20% of mothers of newborns with perinatal HSV have a history of HSV infection All of the following statements regarding (HSV) infections in neonates are true except: A) Most cases are caused by HSV type 2 B) Women with primary HSV genital tract infection are more likely to transmit infection to their offspring than women with recurrent HSV infection C) Most mothers of newborns with perinatal HSV infection have a history of genital HSV infection D) Most mothers of newborns with perinatal HSV infection are asymptomatic at delivery E) Most cases are transmitted at delivery and are not true congenital infections Answer: C Most mothers of newborns with perinatal HSV infection have a history of genital HSV infection Only 15-20% of mothers of newborns with perinatal HSV have a history of obvious HSV infection, and only about 25% have any relevant symptoms at birth All of the following statements concerning the risk of vaccine-associated paralytic poliomyelitis (VAPP) are true except: A-The risk is associated only with OPV and not IPV B-The risk is higher after the second and third doses than after the first dose C-The risk is higher for immunocompromised persons D-The risk is present for household contacts as well as vaccinees E-The risk for VAPP in the United States has exceeded the risk for wild-type polio since 1979 Answer: B The risk is higher after the second and third doses than after the first dose Vaccine-associated paralytic poliomyelitis (VAPP), which follows reversion of the OPV strain to a neuro-virulent strain, has accounted for all cases of polio in the United States since 1979. The risk is higher in vaccinees than in contacts, after the first dose, and in immune- compromised persons. Which of the following features distinguishes paralytic polio from Guillain- Barré syndrome? A-Pleocytosis is uncommon in paralytic polio B-Paralysis is usually asymmetric in paralytic polio C-The paralysis of polio is usually spastic D-Sensory changes are common in paralytic polio E-Paralytic polio only occurs in unimmunized persons Answer: B Paralysis is usually asymmetric in paralytic polio Paralytic polio is characterized by aseptic meningitis accompanied by asymmetric flaccid paralysis without sensory loss. In Guillain-Barré syndrome, the paralysis is characteristically symmetric, and sensory changes (paresthesias) are common. Pleocytosis is common in polio, whereas the cerebrospinal fluid in Guillain-Barré syndrome usually shows only elevated protein and occasionally a few cells. A 14-yr-old boy and his twin sister each developed mumps. Their parents had repeatedly declined for them to receive MMR vaccination. They ask about the prognosis for this disease in their children, especially the possibility of infertility. Which of the following statements most accurately describes the relationship between mumps and infertility? A-Infertility occurs as a sequela of mumps only among prepubertal children B-Infertility is more common among females than among males C-Infertility rate among males with mumps orchitis is approximately 30- 40% D-Infertility among males is rare even with bilateral orchitis E-Infertility rate among females with mumps oophoritis is approximately 30-40% Answer: D Infertility among males is rare even with bilateral orchitis Infertility among males with mumps is rare, even with bilateral orchitis. There is no evidence of impairment of fertility among women with mumps oophoritis. Mumps orchitis and oophoritis are rare among prepubescent boys and girls, and are more common among postpubertal men (14-35%) and women (7%). The most common complication of mumps in childhood is: A-Arthritis B-Meningoencephalitis C-Myocarditis D-Orchitis E-Pancreatitis Answer: B Meningoencephalitis Meningoencephalitis is the most common complication of mumps in childhood, but appears to occur in more than two-thirds of cases, with clinical symptoms in more than 10% of cases Which of the following statements concerning measles and vitamin A is true? A) There is not a confirmed relationship of vitamin A to the prognosis of measles B) Measles causes vitamin A deficiency C) Treatment with vitamin A reduces measles severity for children in developing countries D) Subacute sclerosing panencephalitis (SSPE) is more likely in children with underlying vitamin A deficiency E) Treatment with vitamin A reduces the incidence of SSPE Answer: C Treatment with vitamin A reduces measles severity & complications for children in developing countries There is an apparent correlation between retinol concentration and measles severity. Treatment with vitamin A reduces morbidity and mortality in children with severe measles in developing countries and is recommended for selected children with severe measles in the United States All of the following are characteristic manifestations of congenital rubella syndrome except: A) Snuffles B) Intrauterine growth retardation C) Cataracts D) Structural cardiac defects E) Sensorineural hearing loss. Answer: A Snuffles Congenital rubella affects virtually all organ systems. Snuffles is a sign of congenital syphilis The recommended treatment for active pulmonary tuberculosis in children is: A) Isoniazid for 9 m B) Isoniazid and rifampin for 6 m C) Isoniazid, rifampin, and pyrazinamide for 6-9 m D) Isoniazid and rifampin for 6 m, with pyrazinamide during the first 2 m E) Isoniazid and rifampin for 6 m, with ethambutol during the first 2 mo. Answer: D Isoniazid and rifampin for 6 m, with pyrazinamide during the first 2 mo The American Academy of Pediatrics and the Centers for Disease Control and Prevention recommend that children with pulmonary tuberculosis be treated with 6 m of isoniazid and rifampin supplemented with pyrazinamide for the first 2 m. A previously healthy 6-m-old child who just completed antibiotic for acute otitis media and now is healthy and asymptomatic undergoes a stool culture to check for antibiotic-associated diarrhea. The stool culture grows C. difficile. The preferred antibiotic treatment is: A-Oral clindamycin B-Oral vancomycin C-Intravenous vancomycin D-Oral metronidazole E-None of the above. Answer: E None of the above: antibiotic treatment is not indicated Difficile is frequently isolated from the stool of healthy infants. The interpretation of a positive stool culture or toxin requires clinical correlation. Treatment would not be indicated for an asymptomatic infant. All of the following statements regarding C.difficile-associated diarrhea are true except: A) A-Antibiotic-associated diarrhea is often related to production of a toxin by C.difficile. B) Newborn and young infants are commonly colonized by C.difficile. C) The antibiotic that most commonly causes C.difficile colitis is gentamicin. D) Most children with antibiotic-associated diarrhea will improve without specific antibiotic treatment. E) Treatment of C.difficile colitis should include stopping antibiotics whenever feasible. Answer: C The antibiotic that most commonly causes C.difficile colitis is gentamicin. Virtually all known antibiotics have been implicated; penicillins, broad-spectrum cephalosporins, and clindamycin are the most frequent offenders. A 2-mo-old infant is suspected of having infant botulism. Which is the best means to confirm the diagnosis? A-Lumbar puncture B-CT scan C-Muscle biopsy D-Electromyography E-Fecal specimen Answer: E Fecal specimen The diagnosis of botulism is unequivocally established by demonstrating the presence of botulinum toxin in serum or of C.botulinum toxin or organisms in wound material or feces Which of the following findings on the history or physical examination excludes a diagnosis of infant botulism? A-No ingestion of honey or corn syrup B-Normal latch, suck, and swallow C-Soft feces D-Normal deep tendon reflexes E-Metabolic acidosis Answer: B Normal latch, suck, and swallow In infants, bulbar palsies are evident as poor feeding, weak suck, feeble cry, drooling, and even obstructive apnea Cat-scratch disease classically presents as lymphadenopathy, sometimes accompanied by fever. The most common clinical manifestation of atypical cat-scratch disease is: A-Seizures and coma B-Systemic disease Erythema nodosum C-Oculoglandular syndrome of Parinaud D-Neuroretinitis. Answer: C Oculoglandular syndrome of Parinaud The oculoglandular syndrome (conjunctivitis, lymph node) is the most frequent atypical manifestation of cat-scratch disease. A 12-yr-old boy with cystic fibrosis experiences an acute exacerbation and is admitted for intravenous antibiotic therapy. Sputum culture reveals Pseudomonas aeruginosa. The antibiotic recommended for treatment is: A-Ampicillin-sulbactam B-Ceftazidime C-Cefotetan D-Ceftriaxone E-Ciprofloxacin. Answer: B Ceftazidime Appropriate single agents for treatment of Pseudomonas aeruginosa infection include ceftazidime, cefoperazone, ticarcillin-clavulanate, and piperacillin- tazobactam. Gentamicin or another aminoglycoside may be used concomitantly for synergistic effect. Of the following diagnostic tests, which is least useful in establishing a diagnosis of mesenteric lymphadenitis due to Yersinia pseudotuberculosis? A-Stool culture B-Mesenteric lymph node histology C-Abdominal ultrasonography D-Mesenteric lymph node culture E-Endoscopy Answer: A Stool culture Many patients affected by Y. pseudotuberculosis do not have diarrhea, and thus a stool culture is not even considered as part of the diagnostic evaluation. If the extent of infection is limited to the mesenteric lymph nodes, the stool culture results may be negative. A 26-m-old child with sickle cell disease appears in your office for the first time. He has a history of a prior hospitalization for pneumococcal bacteremia. Currently the child is on penicillin prophylaxis, but he has not received either the conjugate or polysaccharide pneumococcal vaccine. Which of the following is the optimal means for protecting this child from developing another episode of invasive pneumococcal disease? A) A-Give one dose of the pneumococcal conjugate vaccine and continue penicillin prophylaxis. B) Give one dose of the pneumococcal polysaccharide vaccine and continue penicillin prophylaxis. C) Give one dose of the pneumococcal conjugate vaccine followed one month later by one dose of the pneumococcal polysaccharide vaccine, and continue penicillin prophylaxis. D) Give one dose of the pneumococcal conjugate vaccine and switch from penicillin prophylaxis to Augmentin prophylaxis. E) Give one dose of pneumococcal conjugate vaccine and switch to intramuscular benzathine penicillin prophylaxis Answer: C Give one dose of the pneumococcal conjugate vaccine followed one month later by one dose of the pneumococcal polysaccharide vaccine, and continue penicillin prophylaxis. Immunization with the conjugate polysaccharide vaccine is recommended for all infants. High-risk children >2 yr of age, such as those with asplenia, sickle cell disease, some types of immune deficiency (e.g., antibody deficiencies), HIV infection, or chronic lung, heart, or kidney disease (including nephrotic syndrome), may benefit also from the 23-valent pneumococcal polysaccharide vaccine. A 9-yr-old boy presents with fever >39°C for 4 days, myalgias, watery diarrhea, conjunctival infection, diffuse erythroderma, strawberry tongue, blood pressure of 105/45 mm Hg, and moderately elevated hepatic transaminases. The most likely diagnosis is: A-Staphylococcal scalded skin syndrome B-Kawasaki disease C-Toxic shock syndrome D-Stevens-Johnson syndrome E-Toxic epidermal necrolysis. Answer: C Toxic shock syndrome Toxic shock syndrome and Kawasaki disease share many features, but Kawasaki disease is not accompanied by hypotension and shock. Kawasaki disease also typically occurs in children younger than 5 yr. Toxic shock syndrome can complicate focal infections caused by TSST-1-producing strains of Staph. aureus. A 6-m-old child presents with recurrent cellulitis and bacteremia due to Staph. aureus. The white blood cell count is 2500/mm3 with 5% neutrophils, 10% eosinophils, 35% monocytes, and 50% lymphocytes. The platelet count is 650,000/mm3. A brother and a female cousin died at the ages of 18 m and 2 yr, respectively. The most likely diagnosis is: A-AIDS B-Severe combined immunodeficiency C-Kostmann disease D-Cyclic neutropenia E-Chronic granulomatous disease Answer: C Kostmann disease An autosomal recessive severe infantile form of Agranulocytosis, manifests with persistently low absolute neutrophil counts (<200/mm3) and severe, recurrent, and at times lethal (by age 3 Y) infection. All of the following may be associated with neutropenia except: A-Leukocyte adhesion deficiency B-Shwachman-Diamond syndrome C-Cartilage-hair hypoplasia D-Chédiak-Higashi syndrome E-Glycogen storage type Ibdisease Answer: A Leukocyte adhesion deficiency Persons with leukocyte adhesion deficiency have impaired trans-endothelial migration. The circulating neutrophil count with infection is typically above 30,000/mm3 and can surpass 100,000 per mm3, with a paucity of neutrophils in the infected tissues. The mother of an 8-yr-old boy with acute streptococcal tonsillitis calls to report that now, within 15 min after the first dose of oral penicillin V that you prescribed, he is complaining of itching and has developed hives. Which of the following should you recommend? A) A dose of oral Benadryl, with instructions to call again if he has not improved within 30 min B) Immediate return to your office or the nearest emergency department C) Careful monitoring at home, with instructions to return to your office or the nearest emergency department if he becomes short of breath or loses consciousness D) Schedule a visit for a laboratory test to determine serum trypticase level E) Substitution of erythromycin for penicillin Answer: B Immediate return to your office or the nearest emergency department. The urticarial reaction described in the question may develop into anaphylaxis; the latter requires emergency treatment. In addition, the penicillin V should be stopped and a substitute non penicillin antibiotic chosen. Features characteristically associated with atopic dermatitis include all of the following except: A-Allergic rhinitis or asthma B-Elevated serum IgE level C-Peripheral blood eosinophilia D-Lymphopenia Answer: D Lymphopenia Most patients with atopic dermatitis have peripheral blood eosinophilia and elevated serum IgE level. Nearly 80% of patients with atopic dermatitis develop allergic rhinitis and/or asthma A 4-yr-old boy with asthma has had mild wheezing only four times since you began treating him 6 m ago with theophylline twice each day. He previously experienced coughing and wheezing at least three times each week. (A peak serum theophylline concentration 5 m ago was 16 g/mL). For the past 4 days, he has again experienced mild coughing and wheezing responsive to inhaled albuterol. Two days ago, an emergency department physician began treatment with erythromycin & sulfisoxazole for otitis media. This morning he began vomiting. The likely cause of the vomiting is: A-Provocation by coughing (post-tussive emesis) B-Sequelae of otitis media C-Theophylline toxicity D-Albuterol toxicity E-Pediazole intolerance Answer: C Theophylline toxicity The erythromycin (a macrolide antibiotic) component of Pediazole inhibits hepatic theophylline metabolism, thus potentially producing theophylline toxicity. A 7-yr-old boy with asthma has roughness over the extensor surfaces of the upper arms and thighs, which is caused by keratin plugs lodged in the openings of hair follicles. This physical finding is termed: A-Keratosis pilaris B-Fibroepitheliosis C-Hidradenitis D-Xerosis E-Acrochordon Answer: A Keratosis pilaris Xerosis, or dry skin, is the most common skin abnormality of allergic children. Keratosis pilaris, often found on the extensor surfaces of the upper arms and thighs, is characterized by roughness of the skin caused by discrete follicular papules. These are the result of hyperkeratosis with keratin plugs lodged in the openings of hair follicles, and re-form after removal All of the following may be signs of moderate to severe airway obstruction resulting from allergic response except: A-Dennie lines (Dennie-Morgan folds) B-Supraclavicular and intercostal retractions C-Cyanosis D-Pulsus paradoxus E-Respiratory distress with minimal wheezing and a few crackles Answer: A Dennie lines (Dennie-Morgan folds) Dennie lines (Dennie-Morgan folds) are prominent symmetric skinfolds that extend in an arc from the inner canthus beneath and parallel to the lower lid margin. Like allergic shiners and the allergic salute, they are signs of persistent rhinorrhea associated with allergic rhinitis. A "silent chest' in a patient with asthma (answer E) is a severe sign suggesting inspiratory and expiratory obstruction. Cyanosis is always present in such severe cases. Patients with Turner syndrome should undergo careful analysis of their chromosomes for Y chromosome material because they may: A-Become masculinized B-Grow tall C-Become pregnant D-Experience gonadoblastoma E-None of the above. Answer: D Experience gonadoblastoma Y chromosome material is present in 5-10% of girls with Turner syndrome. Gonadoblastoma may develop in the ovary, thus necessitating bilateral oophorectomy as a preventive measure. A mentally retarded 15-yr-old boy is found to have macro-orchidism and large, prominent ears. The most likely diagnosis is: A-Cerebral giantism B-Acromegaly C-Hypothyroidism D-Trisomy 21 E-Fragile X syndrome Answer: E Fragile X syndrome Fragile X syndrome is a common chromosomal cause of mental retardation in boys. Affected boys have allelic expansion of trinucleotide repeats to more than 200 (normal is 6-54 repeats). A disease affecting all of the daughters but none of the sons of an affected father, and 50% of the sons or daughters of an affected mother, is most likely: A-X-linked recessive B-X-linked dominant C-Mitochondrial deletion syndrome D-Autosomal dominant E-Autosomal recessive Answer: B X-linked dominant This is the classic pattern of an X-linked dominant condition. These are rare conditions, which include hypophosphatemic rickets and incontinentia pigmenti (IP). IP is lethal to the male fetus and thus no males are born alive with this disorder. In response to social overtures (being held, hugged, kissed, talked to), a febrile infant does not smile, has a dull, expressionless face, and is not alert to stimuli. The most appropriate approach to management is to: A) A-Administer ceftriaxone IM after a blood culture and have the parent and child return to the office in the morning B) If the child is older than 6 mo, obtain a blood culture and have the parents return to the office if the patient remains febrile C) Administer acetaminophen and reassess after the infant is no longer febrile D) Administer ceftriaxone after obtaining a blood, urine, and CSF specimens for culture and admit the child to the hospital E) Administer a normal saline bolus of 20 mL/kg and reevaluate in 1 hr Answer: D Administer ceftriaxone after obtaining a blood, urine, and CSF specimens for culture and admit the child to the hospital Clinical observation of young patients is critical in helping you to evaluate and distinguish the degree of risk of infection and physiologic impairment. In addition to observing color, tone, grunting, or a bulging fontanel, the response to social stimuli is valuable. This 3-mo-old had pneumococcal meningitis. Indications for admission to the hospital after a burn injury may include all of the following except: A-Suspected child abuse B-Electric burns through an extremity C-Perineal burns D-Poor follow-up E-No tetanus immunization F-Inhalation injury Answer: E No tetanus immunization Lack of immunization against tetanus may be managed with tetanus toxoid and (if a wound is large or dirty) with tetanus immune globulin. The other choices indicate that the patient is at high risk and requires hospitalization. Malignant hyperthermia is associated with all of the following except: A-Hypercarbia B-Masseter spasm C-Metabolic acidosis D-Muscular dystrophy E-Morphine F-A positive family history for the disorder Answer: E Narcotic agents are not associated with the development of this potentially serious and lethal familial condition. The spectrum may range from masseter spasms or a mild increase in intraoperative temperature and CO2 elimination to severe muscle rigidity, hyperpyrexia, metabolic acidosis, and cardiovascular collapse. A 2-yr-old boy is pulseless and apneic after extrication from a swimming pool. Paramedics start CPR, intubate him, and administer 100% oxygen while providing bag-endotracheal tube breaths. An intraosseous line is placed, and two doses of epinephrine are administered. A pulse is found after 10 min of resuscitative measures. The child is transferred to your ED. On arrival, he is deeply comatose with a temperature of 34°C. He is not breathing spontaneously, his heart rate is 120 beats/min, blood pressure is 60/25 mm Hg, and he has cold extremities. Oxygen saturation is 85%. Finger stick glucose is 200 mg/dL. Which of the following interventions is most likely to improve the patient's neurologic outcome? A) Administration of high-dose dextrose to improve cerebral metabolism B) Fluid restriction to minimize potential cerebral edema C) The addition of positive end-expiratory pressure, administration of an isotonic fluid bolus, and an epinephrine infusion to improve oxygenation and circulation D) Keeping the patient hypothermic for the first 12-24 hr of his PICU hospitalization E) Hyperventilation to a PaCO2 of 25 mm Hg Answer: C The addition of positive end-expiratory pressure, administration of an isotonic fluid bolus, and an epinephrine infusion to improve oxygenation and circulation Despite many hopeful new interventions for hypoxic-ischemic neurologic injury, none has been proven beneficial. The best approach is to support normal oxygenation and normal blood pressure to avoid continued cerebral hypoxia and ischemia, respectively. Monitor blood pressure to improve cerebral perfusion pressure and possibly to avoid secondary ischemic injury from raised intracranial pressure A 6-yr-old child with meningococcal meningitis develops a swollen left knee on the fifth day of antibiotic treatment. Which of the following is the most likely etiology of this finding? A) A-Hemorrhage into the joint occurring as a result of disseminated intravascular coagulation B) Progression of septic arthritis that was unrecognized at the onset of illness C) Nonspecific edema from progressive sepsis-related capillary leak D) Immune complex deposition resulting from production of anti-meningococcal antibodies E) Secondary bacterial infection from the immunosuppressive effects of meningococcal infection. Answer: D Immune complex deposition resulting from production of anti-meningococcal patients with meningococcal disease showed one or more of the three allergic complications: arthritis, cutaneous vasculitis, and episcleritis. These complications, which were often multiple, occurred six to nine days after the beginning of the illness and three to six days after the start of successful antibiotic therapy. Those patients with severe systemic disease were prone to the complications. Histological and bacteriological study of the arthritis and vasculitis showed that these lesions were probably not due to persisting infection and suggested that they might be due to immune complex disease. A 6-month-old infant has been exclusively fed a commercially available infant formula. Upon introduction of fruit juices, however, the child develops jaundice, hepatomegaly, vomiting, lethargy, irritability, and seizures. Tests for urine reducing substances are positive. Which of the following is likely to explain this child’s condition? a. Tyrosinemia b. Galactosemia c. Hereditary fructose intolerance d. α1-Antitrypsin deficiency e. Glucose-6-phosphatase deficiency Answer: C The patient in the question likely has “Hereditary Fructose Intolerance”, manifest only when fructose in fruit juice is provided in the diet. Galactosemia, fructosemia,tyrosinosis, and glucose-6-phosphatase deficiency represent diseases in which a congenital deficiency of enzyme causes an interruption of a normal metabolic pathway and an accumulation of metabolic precursors that damage vital organs. Galactose (found in milk) and fructose (found in fruit juices) produce urinary-reducing substances in their respective disorders. The mode of inheritance of galactosemia, fructosemia, and most forms of glucose-6-phosphatase deficiency is autosomal recessive. In Galactosemia and Fructosemia, errors in carbohydrate metabolism cause the accumulation of toxic metabolites when specific dietary sugars are introduced (lactose in galactosemia; fructose and sucrose in fructosemia). Exclusion of the offending carbohydrate from the diet will prevent liver damage. In Tyrosinemia type I, or Tyrosinosis, the accumulation of tyrosine and its metabolites is associated with severe involvement of the liver, kidney, and CNS. Manifestations of acute liver failure can appear in infancy. A chronic form of the disorder presents as progressive cirrhosis and leads to liver failure or hepatoma. Dietary management does not prevent liver disease. Glucose-6-phosphatase deficiency often presents at 3 to 4 months of age with failure to thrive, hypoglycemia, hepatomegaly, and acidosis. α1-Antitrypsin deficiency causes liver disease through accumulation of an abnormal protein, caused by a single amino acid substitution on chromosome 14. It has a variable presentation, but the following are common in infancy: cholestasis; bleeding into the CNS, gastrointestinal (GI) tract, or at the umbilical stump; and elevation of transaminase concentrations. Affected children may have chronic hepatitis with cirrhosis and portal hypertension. An 18m old child presents to the emergency center having a brief, generalized tonic-clonic seizure. He is now postictal and has a temperature of 40°C (104°F). During the lumbar puncture (which ultimately proves to be normal), he has a large, watery stool that has both blood and mucus in it. Which of the following is the most likely diagnosis in this patient? a. Salmonella b. Enterovirus c. Rotavirus d. Campylobacter e. Shigella.. Answer: E Clinical manifestations of shigellosis range from watery stools for several days to severe infection with high fever, abdominal pain, and generalized seizures. In addition, there is a rare and fatal “toxic encephalopathy” seen with Shigella infection known as Ekiri syndrome. In general, about 50% of infected children have emesis, greater than two-thirds have fever, 10% to 35% have seizures, and 40% have blood in their stool. Often, the seizure precedes diarrhea and is the complaint that brings the family to the physician. Fever usually lasts about 72 hours, and the diarrhea resolves within 1 to 2 weeks. Presumptive diagnosis can be made on the clinical history; confirmation is through stool culture. Supportive care, including adequate fluid and electrolyte support, is the mainstay of therapy. Antibiotic treatment is problematic; resistance to trimethoprim sulfamethoxazole and ampicillin is common, necessitating therapy with third-generation cephalosporins in many cases. As always, knowledge of the susceptibility patterns of the bacteria in your area is the key to using the right antibiotic. A previously healthy 5-year-old boy has a 1-day history of low-grade fever, colicky abdominal pain, and a rash. He is well appearing and alert. His vital signs, other than a temperature of 38°C (100.5°F), are completely normal. A diffuse, erythematous, maculopapular, and petechial rash is present on his buttocks and lower extremities, as shown in the photograph. He has no localized abdominal tenderness or rebound; bowel sounds are active. Laboratory data demonstrate: Urinalysis:.30 red blood cells (RBCs) per high-powered field, 2+ protein Stool: Guaiac positive Platelet count: 135,000/μL These findings are most consistent with which of the following? a. Anaphylactoid purpura b. Meningococcemia c. Child abuse d. Leukemia e. Hemophilia B Answer: A The clinical presentation described supports the diagnosis of Anaphylactoid purpura (also known as Henoch-Schönlein purpura, or HSP), a generalized, acute vasculitis of unknown cause involving small blood vessels. In this condition, the skin lesion, which is classic in character (palpable purpura) and distribution, is often accompanied by arthritis, usually of the large joints, and by gastrointestinal symptoms. Colicky abdominal pain, vomiting, and melena are common. Renal involvement occurs in a significant number of patients and is potentially the most serious manifestation of the disease. Although most children with this complication recover, a few will develop chronic nephritis. Laboratory studies are not diagnostic. The platelet count, serum complement, and IgA levels can be normal or elevated. Coagulation studies and platelets are normal. Meningococcal infection and leukemia should be in the differential diagnosis, as both can cause purpura, but in a well-appearing child with normal vital signs and normal blood counts they are unlikely. Child abuse and hemophilia will typically result in bruises, not petechiae. Two weeks after a viral illness, a 9-year-old girl presents to your clinic with a complaint of several days of weakness of her mouth. In addition to the drooping of the left side of her mouth, you note that she is unable to completely shut her left eye. Her smile is asymmetric, but her examination is otherwise normal. Which of the following is the most likely diagnosis? a. Guillain-Barré syndrome b. Botulism c. Cerebral vascular accident d. Brainstem tumor e. Bell palsy Answer: E Bell’s palsy Is an acute, unilateral facial nerve palsy that begins about 2 weeks after a viral infection. The exact pathophysiology is unknown, but it is thought to be immune. On the affected side, the upper and lower face are typically paretic, the mouth droops, and the patient cannot close the eye. Treatment consists of maintaining moisture to the affected eye (especially at night) to prevent keratitis. Complete, spontaneous resolution occurs in about 85% of cases, 10% of cases have mild residual disease, and about 5% of cases do not resolve. You are asked to review a child on the ward who is known to have short stature and renal abnormalities. On examination, she has micrognathia and an ejection systolic murmur at the upper left sternal edge. Her notes show that she has recently seen an ophthalmologist. What is the MOST likely underlying diagnosis? A. Williams syndrome B. DiGeorge syndrome C. Alagille syndrome D. Noonan syndrome E. Left atrial isomerism. Answer: C Alagille syndrome Alagille syndrome is a genetic defect of the JAG-1 gene in 70% of cases. Features include peripheral pulmonary artery stenosis, a prominent forehead, wide- apart eyes, small chin, butterfly vertebrae, intrahepatic biliary hypoplasia, embryotoxon (slit lamp for cornea), and renal and growth abnormalities. Posterior embryotoxon occurs when there is a prominent Schwalbe’s line visible just inside the temporal limbus. It occurs in approximately 15% of normal eyes and is visible through a clear cornea as a sharply defined, concentric white line or opacity anterior to the limbus. Williams syndrome is due to a 1.5-Mb deletion on chromosome 7 and leads to typical facial features, behavioural abnormalities and cardiac features of supravalvar aortic stenosis and branch pulmonary artery stenosis. DiGeorge syndrome is associated with conotruncal defects (tetralogy of Fallot, common arterial trunk and interrupted aortic arch), typical facial features, cleft palate, absent thymus and absent parathyroids. Noonan syndrome is associated with mutations in PTPN11, SOS1, KRAS or RAF1 genes, with cardiac features of hypertrophic cardiomyopathy, atrial septal defect, pulmonary stenosis and pulmonary hypertension. You are asked to see a 6-year-old girl with a diagnosis of right atrial isomerism. Which one of the following features would you expect her to have? A. Asplenia and a midline liver B. Polysplenia C. Two functional left lungs D. T-cell deficiency E. Trisomy 21 Answer: A Asplenia and a midline liver Right atrial isomerism is a multifactorial genetic defect associated with asplenia, small-bowel malrotation, and complex heart disease with abnormalities of connection, in which the pulmonary veins always connect abnormally because there is no morphological left atrium with which to connect. In Left atrial isomerism, there is polysplenia, small-bowel malrotation (less common than in right atrial isomerism), two left lungs and complex heart disease. The majority of cases present at the time of birth or within a few days or weeks with clinical features of the congenital heart defect (which may including symptoms of cyanosis, breathless, lethargy and poor feeding. Prenatal diagnosis can be done by Echocardiography. You are asked to review an ECG of a baby on the intensive care unit. The baby was well at birth, but soon became unwell and cyanosed. There was no heart murmur. ECG findings reveal a superior axis, absent right ventricular voltages, and a large P wave. What is the MOST likely diagnosis? A) Complete atrioventricular septal defect B) Tricuspid atresia C) Critical pulmonary stenosis D) Transposition of the great arteries E) Total anomalous pulmonary venous connection (TAPVC). Answer: B Tricuspid atresia Tricuspid atresia is the condition in which there is no tricuspid valve and usually the right ventricle is very small. There is right-to-left shunt at atrial level, as the blood cannot pass into the right ventricle. Babies become very cyanosed when the ductus arteriosus closes if they are duct- dependent, and they usually have no heart murmur. Management options include a Blalock–Taussig shunt if the child is very blue, a pulmonary artery (PA) band if they are in heart failure, a hemi-Fontan procedure after they reach 6 months of age, and a Fontan procedure at 3–5 years of age. The ECG shows a superior axis, as the atrioventricular junction is located inferiorly, the P waves are large as a result of right atrial hypertension, and there is a small right ventricle, reducing the forces visible on the ECG. Scimitar syndrome: A) Is usually associated with a hypoplastic left lung B) Can be palliated by coil occlusion in the cardiac catheter laboratory C) Will show dextrocardia on the X-ray as a result of situs inversus D) Is associated with abnormal pulmonary arterial supply E) Is usually associated with abnormal radii. Answer: B,D Scimitar syndrome is a form of anomalous pulmonary venous drainage in which the veins from the lower right lung drain into the inferior vena cava. The right lung itself is hypoplastic, and there is an associated dextrocardia due to the heart moving over to the right side of the chest, but with normal situs. Situs is the orientation of the organs, situs solitus being normal, and situs inversus being mirror image. The arterial supply to the lung is from branches of the descending aorta. The right upper lobe pulmonary vein draining into the inferior vena cava may be seen as a vertical line on a chest X-ray and is known as the ‘Scimitar sign’. There may be an atrial septal defect, and children can suffer with recurrent chest infections, which may require right lower lobectomy. A 3-month-old African American boy presented to the dermatology clinic for evaluation of scalp lesions that were not improving with topical or oral antifungal medications. These lesions were present at birth but enlarged by 2 weeks of age. The infant had no other systemic symptoms. Maternal history was significant for mixed connective tissue disease. Physical examination revealed a thriving infant with 2 erythematous plaques with well-demarcated borders and central atrophy on the left parietal scalp and postauricular region. The remainder of his physical examination was unremarkable. Laboratory studies revealed an elevated AST level of 339 U/L (reference range, 10-60 U/L) and ALT level of 364 U/L (reference range, 5-50 U/L) but an otherwise normal complete blood cell count and normal comprehensive metabolic profile. What is your diagnosis? a) Neonatal candidiasis capitis b) Neonatal lupus erythematosus c) Neonatal tinea capitis d) Neonatal scleroderma Answer: B NEONATAL LUPUS ERYTHEMATOSUS AND ASSOCIATED HEPATITIS. Based on the cutaneous findings and positive antibodies to Sjögren syndrome antigen A/Ro (SSA/Ro) and Sjögren syndrome antigen B/La (SSB/La), a diagnosis of neonatal lupus erythematosus (NLE) was established. Results of scalp fungal culture were negative. Electrocardiogram (ECG) and echocardiogram were normal. Photoprotection was recommended and use of hydrocortisone ointment, 2.5%, was started twice daily for 2 weeks. The scalp lesions resolved by 6 months of age with no sequelae. No further manifestations of liver insufficiency developed, and hepatic transaminase levels normalized by 8 months of age. The patient's mother had been diagnosed with mixed connective tissue disease 3 years prior, but her antibody status was uncertain The syndrome is acquired by the transplacental passage of antibodies to SSA/Ro and/or SSB/La extractable nuclear antigens. These antibodies are often found in patients with known rheumatic disease, such as systemic lupus erythematosus and Sjögren syndrome. Neonatal lupus erythematosus is an autoimmune syndrome that may have cutaneous, cardiac, hematologic, and hepatic manifestations. An 18-month-old child has been brought to your urgent care clinic for evaluation. His mother tells you that she found him playing with an open bottle of his grandmother's medication. On physical examination, he is sleepy but arousable, pale, mildly diaphoretic, and afebrile. His respiratory rate is 20 breaths/min, heart rate is 60 beats/min, and blood pressure is 65/40 mm Hg. His lungs are clear, there are no murmurs, and his pulses are weak. The MOST likely cause for this patient's presentation is ingestion of A. beta blocker B. captopril C. hydralazine D. pseudoephedrine E. tricyclic antidepressant... Answer: A Accidental ingestion of medications continues to be a significant cause of pediatric morbidity and potential mortality. The problem is greatest in toddlers and is exacerbated when medications are left within reach of children or not appropriately secured in a locked cabinet. The reaction to ingestion of a given medication depends on the class and dose. Beta blockers: Can have varying effects on the heart and other organ systems. In the heart, they typically exhibit some degree of negative chronotropic (slowing of the heart rate), dromotropic (slowing of the conduction through the AV node), and inotropic (decrease in the ventricular force of contraction) effects. As a result, this class of medications generally is used to control certain types of arrhythmia and hypertension and to reduce myocardial work and oxygen demand. The depressed sensorium, bradycardia for age, and hypotension with diminished pulses described for the child in the vignette are most consistent with beta blocker ingestion. Captopril: is an angiotensin-converting enzyme (ACE) inhibitor. ACE activity converts angiotensin I to angiotensin II, which is a potent vasoconstrictor. Therefore, ACE inhibitors lower the systemic vascular resistance and the systemic blood pressure, decreasing the afterload (work) of the left ventricle. Hydralazine: is an arteriolar dilator that may have some positive inotropic effect. It often is used as antihypertensive therapy and typically increases the heart rate. Pseudoephedrine is an alpha agonist that is used commonly for relief of symptoms caused by nasal congestion. Its effects on the cardiovascular system are to cause systemic vasoconstriction that may increase blood pressure, and it also may cause tachycardia. Tricyclic antidepressants: are a diverse group of medications that can be used for a variety of noncardiac indications, including depression. When taken in high doses, they may affect the cardiovascular system by causing vasodilation, flushing, hypotension, and tachyarrhythmias. Although captopril, hydralazine, and tricyclic antidepressants may be associated with hypotension, they all would be expected to cause (or be associated with) a corresponding tachycardia. All the following conditions are associated with an increased incidence of cancer EXCEPT: A. Down syndrome B. Fanconi’s anemia C. von Hippel–Lindau syndrome D. Neurofibromatosis E. Fragile X syndrome. Answer: E A small proportion of cancers occur in patients with a genetic predisposition. Roughly 100 syndromes of familial cancer have been reported. Recognition allows for genetic counseling and increased cancer surveillance. Down syndrome: is characterized clinically by a variety of features, including moderate to severe learning disability, facial and musculoskeletal deformities, duodenal atresia, congenital heart defects, and an increased risk of acute leukemia. Fanconi’s anemia: is a condition that is associated with defects in DNA repair. There is a higher incidence of cancer, with leukemia and myelodysplasia being the most common cancers. Von Hippel–Lindau syndrome: is associated with hemangioblastomas, renal cysts, pancreatic cysts and carcinomas, and renal cell cancer. Neurofibromatosis (NF) types I and II are both associated with increased tumor formation. NF II is more associated with schwannoma. Both carry a risk of malignant peripheral nerve sheath tumors. Fragile X: is a condition associated with chromosomal instability of the X chromosome. These patients have mental retardation; typical morphologic features, including macro-orchidism and prognathia; behavioral problems; and occasionally seizures. Increased cancer incidence has not been described. Fragile X syndrome, also termed Martin-Bell syndrome or marker X syndrome, is the most common cause of inherited mental retardation, intellectual disability, and autism and is the second most common cause of genetically associated mental deficiencies, after trisomy 21. Conservative estimates are that fragile X syndrome affects approximately 1 in 2500-4000 males and 1 in 7000-8000 females. All of the following conditions can cause optic neuritis except: 1-Devic disease 2-Lead poisoning 3-Vincristine therapy 4-Corticosteroids 5-Bacterial meningitis .. Answer: 4 (Corticosteroids) Intravenous corticosteroid therapy can improve a visual recovery in young adults but does not alter visual outcome. Oral corticosteroid should not be used because of increased recurrence rate of optic neuritis. Neuromyelitis optica (Devic disease) can cause bilateral optic neuritis. A 5-year-old boy appears with an abrupt onset of increased urinary frequency for the last 3 days. He has been voiding every 10-15 minutes during the day. The mother denies history of fever, abdominal pain, dysuria, nocturia, or daytime incontinence. The urinalysis result is normal. The next step in management is: 1-Trimethoprim-sulfamethoxazole 2-Restriction of fluid intake 3-Renal ultrasonography 4- 24-hour urine sample for calcium 5-Reassurance. Answer: 5 (Reassurance) The condition is self-limited and symptoms resolve within 2-3 months. Rarely, anticholinergic therapy is effective. This boy is diagnosed with pollakiuria or daytime frequency syndrome of childhood. This may be due to stress for an unknown reason or before the onset of kindergarten All of the following conditions can present with spherocytosis in peripheral blood smear except: 1-ABO incompatibility 2-Thermal injury 3-Wilson disease 4-Autoimmune hemolytic anemia 5- Pneumococcal sepsis. Answer: 5 (Pneumococcal sepsis) Pneumococcal sepsis does not cause spherocytosis. However, clostridial septicemia with exotoxemia can cause spherocytosis. Your 13 month old had a high fever for 4 days and then broke out in a red rash all over once the fever broke. He likely has. a) Impetigo b) Herpes c) Chickenpox d) Roseola e) Robella f) Measles Answer: D (Roseola) Although all of these conditions cause a rash and are caused by infections, only roseola has the typical rash that begins once the fever breaks. Two common and closely related viruses can cause roseola: human herpesvirus (HHV) type 6 and possibly type 7. These viruses belong to the same family as the better-known herpes simplex viruses (HSV). A 2 year old girl presented with a 12-hour history of fever and poor feeding. The family owned two cats. Her temperature was 39.5°C, and she was irritable, with no localising signs or skin lesions. A full septic screen was performed. Cerebrospinal fluid (CSF) showed a neutrophilic pleocytosis and gram-negative coccobacilli. She was treated with intravenous cefotaxime and gentamicin. Within 24 hours both CSF and blood cultures showed growth of gram-negative bacilli. What is the likely infective organism? A. Legionella pneumophilia B. Haemophilus influenzae C. Pasteurella multicoda D. E coli E. Pseudomonas aeruginosa. Answer: C (Pasteurella multicoda) Pasteurella multicoda is an oral commensal of domestic pets known to be an opportunistic human pathogen after traumatic animal contact. The most common infections in humans are skin and pulmonary infections. Pasteurella meningitis occurs at extremes of age (infants), in the immune- compromised (associated with liver cirrhosis, renal disease and haematological malignancies) and after traumatic head injury. . A 12-yr-old boy with cystic fibrosis experiences an acute exacerbation and is admitted for intravenous antibiotic therapy. Sputum culture reveals Pseudomonas aeruginosa. The antibiotic recommended for treatment is: A-Ampicillin-sulbactam B-Ceftazidime C-Cefotetan D-Ceftriaxone E-Ciprofloxacin Answer: B Ceftazidime Appropriate single agents for treatment of Pseudomonas aeruginosa infection include ceftazidime, cefoperazone, ticarcillin-clavulanate, and piperacillin- tazobactam. Gentamicin or another aminoglycoside may be used concomitantly for synergistic effect. Which of the following indicates a poor prognosis for cholera in children? A- Illness complicated by hypokalemia B- Acidosis C- Acute tubular necrosis D- Tachycardia and tachypnea E- Hypoglycemia and seizures Answer: E Hypoglycemia and seizures Explanation: Coma and death are more common with cholera that is complicated by seizures associated with hypoglycemia. In one study, 14.3% of children with cholera complicated by hypoglycemia died, compared with 0.7% of children without hypoglycemia. After dehydration, hypoglycemia is the most common life-threatening consequence of cholera in children. A 2 month old baby is admitted with seizures and failure to thrive. Follwoing a seizure he is found to have a blood glucose concentration of 1.2 mmol/l. Other biochemistry reveals a lactic acidosis and hyperlipidaemia. Examination reveals hepatomegaly with palpable kidneys. What is the most likely diagnosis? 1) Gaucher's disease 2) McArdle's disease 3) Niemann-Pick disease 4) Pompe Disease 5) Von Gierke disease Answer: 5 (Von Gierke disease) This baby has hypoglycaemia accounting for fits, associated with lactic acidosis and hepatomegaly which would point to a diagnosis of von Gierke's disease. This is a glycogen storage disease due to deficiency of glucose-6-phosphatase. Glycogen is stored in both liver and kidney causing enlargement. Both Gaucher and Niemann-Pick disease are lipid storage disorders. McArdle's disease is a glycogen storage disease affecting muscle rather than liver as is Pompe's disease. Neither would be expected to produce profound hypoglycaemia. A 10-yr-old boy spilled gasoline on his legs. His pants became ignited and he suffered 20% body surface area burns. The most important treatment is: A-7-day course of penicillin B-5-day course of penicillin C-10-day course of penicillin D-None of the above Answer: D None of the above Initial treatment is to remove the clothing and place warm saline dressings over the wound. Penicillin is not an immediate therapy, and there is controversy about whether it should be used at all.
Factors Affecting Utilization of Intermittent Preventive Treatment of Malaria Among Pregnant Mothers Attending Antenatal in Kabuyanda Health Centre IV-Isingiro District